Esercizio sul bilanciamento delle forze in meccanica

Messaggioda Stefano.saloma » 23/01/2020, 06:50

Oggi vi presento un punto di un esercizio che sembra semplice, ma mi risulta altamente ostico :
Immagine
Si consideri un blocco di massa M=1.35 kg posto su un piano orizzontale scabro con
coefficiente di attrito dinamico µD=0.250 a cui è applicata una forza costante F che forma
con l'orizzontale un angolo $\theta $ =40 gradi
, cosicché la forza F (di modulo 10.7 N) ha una
componente diretta verso il basso. Al di sopra del blocco M viene posto un oggetto di
massa m=419 g di dimensioni trascurabili, che può essere considerato puntiforme. Tra il
punto materiale m ed il blocco M vi è attrito. In tali condizioni l'attrito fa sì che non vi sia
moto relativo tra i due corpi. Se i due corpi sono inizialmente fermi, calcolare:
l'accelerazione con cui i due corpi si muovono
Allora io per risolverlo ho provato in più modi diversi e quello che si avvicina di più al risultato di 1.21 ms^-2 è :
$$ \sum F_{y} = 0 quindi: - Fsin(\theta) - (M+m) g - (m+M) asin(\theta) + R = 0$$ (dove R è la reazione vincolare), mentre $$ \sum F_{x} = (M+m) acos(\theta) quindi : Fcos(\theta) - \mu _{d} R = (M+m) a cos (\theta)$$
Dove ho sbagliato? In caso avessi sbagliato, come andrebbe fatto? Grazie in anticipo
Stefano.saloma
Starting Member
Starting Member
 
Messaggio: 10 di 34
Iscritto il: 04/08/2019, 23:15

Re: Esercizio sul bilanciamento delle forze in meccanica

Messaggioda Stefano.saloma » 23/01/2020, 07:52

Wait sorry ho trovato il mio errore, andava bene cone avevo scritto prima, cioè la somma forza in y uguale 0 (senza la castroneria del $(m+M) a sin(\theta) $ e la somma forze in x uguale a (M+m)a
Stefano.saloma
Starting Member
Starting Member
 
Messaggio: 11 di 34
Iscritto il: 04/08/2019, 23:15

Re: Esercizio sul bilanciamento delle forze in meccanica

Messaggioda Lucacs » 23/01/2020, 11:21

Il problema sono in realtà tre problemi in uno.
1) la forza non basta a spostarli $F<=mu_1(m_1+m_2)g=F_((1-2) at) $
2) i corpi si muovono con accelerazioni diverse
3)i corpi si muovono con la stessa accelerazione $a_1=a_2=(F-F_((1-2) at)) /(m_1+m_2)$
Quando $ (mu_1+mu_2)/(m_1+m_2)g>F>F_((1-2)at) $

Veniamo al punto 2:
$m_1: F-mu_1(m_1+m_2)g-mu_2m_2g=m_1a_1$
$m_2:mu_2m_2g=m_2a_2$
Risulta quindi $a_1>a_2 $ se si ha $ F>(mu_1+mu_2)(m_1+m_2)g$$>F_((1-2)at) $
Se risolvi il sistema trovi $a_2=mu_2g$ costante e $a_1$ variabile
Lucacs
Average Member
Average Member
 
Messaggio: 42 di 980
Iscritto il: 06/01/2020, 10:07


Torna a Fisica, Fisica Matematica, Fisica applicata, Astronomia

Chi c’è in linea

Visitano il forum: Nessuno e 1 ospite